Dadas las matrices de densidad de los subsistemas, ¿es posible en general recuperar la matriz de densidad de todo el sistema?

Consideremos un sistema bipartito en un estado mixto entrelazado. Dado que su matriz de densidad siempre se puede diagonalizar, podemos escribirla de las siguientes maneras:

(1) ρ A B = j pag j | ψ j ψ j | = i , k C i , k | a i a i | | b k b k | ,

dónde { | ψ j } es una base ortonormal en H A B = H A H B , y bases ortonorales en H A y H B son dados por { | a i } y { | b j } .

Se pueden obtener matrices de densidad reducida para los subsistemas A y B:

(2) ρ A = T r B ( ρ A B ) = i , k C i , k | a i a i | ρ B = i , k C i , k | b k b k |

Entonces mi pregunta es: dados los espectros (conjuntos de valores propios) de ρ A y ρ B , es en general posible recuperar el espectro de ρ A B ? Si no, ¿hay alguna intuición física detrás de este resultado? ¿Se puede generalizar este resultado para cualquier sistema multipartito?

Como modelo de juguete consideré d i metro ( H A ) = 2 ; y d i metro ( H B ) = 3 . Una vez aplicado (1) y (2) y teniendo en cuenta que las trazas de todas las matrices de densidad = 1 , he llegado a un sistema de ecuaciones con un número de incógnitas mayor que el número de ecuaciones independientes. Entonces, a menos que me falten otras restricciones, esta tarea parece no tener solución en general.

EDITAR:

Quiero arreglar puntos mencionados por Emilio Pisanty y Luzanne.
(Al menos) una cosa que he pasado por alto es que para una base ortonormal arbitraria { | ψ j } expresión ρ A B = j pag j | ψ j ψ j | no tiene que estar dado por una matriz diagonal con valores propios pag i en su diagonal. Sin embargo (como creo) siempre existe tal base ortonormal que satisface la condición antes mencionada. Por ejemplo podemos tomar por | ψ k columna con solo k-ésimo elemento distinto de cero e igual a 1.
Ahora definimos bases ortonormales { | a i } y { | b j } exactamente de la misma manera. Entonces (me parece) tenemos la siguiente conexión:

| ψ j = C i , k j | a i | b k
donde no me refiero a la suma sobre i y k . C i , k j = 1 para cada combinación de i , k y j . Para esta elección de bases altamente específica (1) debería ser válida.

Aún así, insinúo que mi razonamiento sobre la recuperación de ρ A B de ρ A y ρ B debería aguantar porque:

  • conjunto de valores propios no depende de la elección de la base
  • la matriz de densidad está completamente (?) definida por un conjunto de valores propios
physics.stackexchange.com/questions/1491/… esta pregunta parece estar relacionada con la mía, pero me parece que el autor analiza algunos otros aspectos.
tu expresión en ( 1 ) es extremadamente dudoso y, por lo menos, no obvio. Qué te hace pensar que ρ A B se puede descomponer de esa manera?
Sospecho que el lado derecho de (1) debería ser i , i , k , k C i , i , k , k | a i a i | | b k b k | , como sigue de la escritura | ψ j = i , k C i , k j | a i | b k y ψ j | = i , k C i , k j , a i | b k | . Y, por supuesto, tal descomposición no es única... (podemos elegir cualquier base ortonormal ( a i ) i y ( b k ) k queremos)
@Luzanne Bueno, es difícil cerrar la puerta por completo a la existencia de alguna base especial que eliminará parte de la complejidad de su expresión general. (Pero en este caso no creo que sea factible, o al menos no por la expresión de Yaroslav).
@EmilioPisanty Si ( a i ) i y ( b k ) k son bases ortonormales como se afirma, entonces (1) tal como está implicaría que el | a i | b k formar una base de estados propios para ρ . Esto es demostrablemente imposible, por ejemplo, para | Ψ Ψ | (usando la notación de su respuesta), bcs el espacio propio para el valor propio 1 es el espacio unidimensional Durar { | Ψ } que no se puede escribir como el lapso de un número de | a i | b k .
@Luzanne Estaba pensando en términos de elegir bases propias locales para las matrices de densidad reducida, lo que posiblemente podría simplificar las cosas. (A menudo se les llama "orbitales naturales" en contextos de química cuántica). Pero estoy de acuerdo, no funcionará aquí.

Respuestas (2)

Dejando de lado algunos aspectos dudosos de su pregunta (en particular, el hecho de que su descomposición en ( 1 ) es casi seguro que no es posible en general), hay una respuesta fácil a la pregunta central que plantea:

Entonces mi pregunta es: dados los espectros (conjuntos de valores propios) de ρ A y ρ B , es en general posible recuperar el espectro de ρ A B ?

No , esto no es posible. Esto es fácil de ver comparando

  • ρ A B ( 1 ) = | Ψ Ψ | dónde | Ψ es un estado entrelazado al máximo en dos qubits, y
  • ρ A B ( 2 ) = 1 4 I 4 , el estado de máxima mezcla de dos qubits.

Ambos tienen matrices de densidad reducida idénticas, ρ A = ρ B = 1 2 I 2 con valores propios ( 1 , 1 ) , pero el espectro de ρ A B ( 1 ) es ( 1 , 0 , 0 , 0 ) y el espectro de ρ A B ( 2 ) es ( 1 4 , 1 4 , 1 4 , 1 4 ) .

Para complementar el claro contraejemplo de Emilio Pisanty y tratar de responder a la

Si no, ¿hay alguna intuición física detrás de este resultado?

parte de la pregunta, vale la pena señalar que lo mismo ya es cierto en la física estadística clásica .

Si tengo una distribución de probabilidad pag ( a , b ) para el estado de un sistema clásico compuesto, no es posible reconstruir esta distribución de probabilidad conjunta a partir de las distribuciones marginales pag 1 ( a ) = pag ( a , b ) d b y pag 2 ( b ) = pag ( a , b ) d a . Lo que la distribución conjunta pag codifica que no es capturado por pag 1 y pag 2 solo, es el conocimiento de cómo el estado del sistema 1 se correlaciona con la del sistema 2 .

En el caso cuántico, lo que se pierde si solo conocemos las matrices de densidad parcial para cada sistema son las correlaciones tanto clásicas como cuánticas entre los dos sistemas.